Administración     

Olimpiadas de Matemáticas
Página de preparación y problemas

Selector
La base de datos contiene 1154 problemas y 775 soluciones.
OME Local
OME Nacional
OIM
OME Andalucía
Retos UJA
Problema 704
¿Existe algún triángulo tal que las medidas de sus lados son tres números enteros consecutivos y el ángulo mayor es el doble que el menor? Si existe, determinar sus medidas.
pistasolución 1info
Pista. Fíjate en que los lados están ordenados igual que sus ángulos opuestos. Utiliza los teoremas del seno y el coseno para obtener relaciones entre los lados y los ángulos.
Solución. Pongamos que los lados miden $n-1$, $n$ y $n+1$ y que el ángulo menor es $\alpha$ y el mayor $2\alpha$. Esto nos dice que $2\alpha$ es el ángulo opuesto a $n+1$ y $\alpha$ el opuesto a $n-1$ ya que los ángulos guardan el mismo orden que sus lados opuestos. Por tanto, el teorema del seno nos dice que \[\frac{\mathrm{sen}(\alpha)}{n-1}=\frac{\mathrm{sen}(2\alpha)}{n+1}=\frac{2\mathrm{sen}(\alpha)\cos(\alpha)}{n+1}\ \Longrightarrow\ \cos(\alpha)=\frac{n+1}{2(n-1)}.\] Ahora bien, el teorema del coseno aplicado al lado de longitud $n-1$ nos dice que \[(n-1)^2=n^2+(n+1)^2-2n(n+1)\cos(\alpha)=2n^2+2n+1-\frac{n(n+1)^2}{n-1}.\] Operando y simplificando, la ecuación anterior equivale a $n(n-5)=0$, lo que nos da como única posibilidad $n=5$ (el triángulo de lados $4$, $5$ y $6$).

Resta por ver si este triángulo cumple la propiedad. De nuevo por el teorema del coseno, tenemos que el ángulo $\alpha$ opuesto al lado de longitud $4$ cumple \[\cos(\alpha)=\frac{5^2+6^2-4^2}{2\cdot 5\cdot 6}=\frac{3}{4}.\] Por su parte, el ángulo $\beta$ opuesto al lado de longitud $6$ cumple \[\cos(\beta)=\frac{4^2+5^2-6^2}{2\cdot 4\cdot 5}=\frac{1}{8}=2\cos^2(\alpha)-1=\cos(2\alpha).\] Deducimos que $\beta=2\alpha$, luego en este triángulo el ángulo mayor es el doble del menor y respondemos así afirmativamente a la pregunta del enunciado.

Si crees que el enunciado contiene un error o imprecisión o bien crees que la información sobre la procedencia del problema es incorrecta, puedes notificarlo usando los siguientes botones:
Informar de error en enunciado Informar de procedencia del problema
Problema 703
Calcular los posibles valores de $f(2004)$, donde $f:\mathbb{N}\to\mathbb{N}$ es una función que cumple las siguientes tres condiciones:
  • $f(mn)=f(m)f(n)$ para todo $m,n\in\mathbb{N}$,
  • $f(n)\leq n^2$ para todo $n\in\mathbb{N}$,
  • $f(1002)\geq 1003969$.
pistasolución 1info
Pista. Observa que $1002=2\cdot 3\cdot 167$ y, por tanto, $f(1002)=f(2)f(3)f(167)$. Usa esto para hallar los valores de $f(2)$, $f(3)$ y $f(167)$.
Solución. La primera condición nos dice que la función $f(n)$ se lleva muy bien con la factorización de $n$, luego factorizamos $1002=2\cdot 3\cdot 167$ ($167$ es primo, pero no hace falta ni siquiera saberlo para llegar al resultado), lo que nos da $2004=2^2\cdot 3\cdot 167$. Ahora observamos que \[1003969\leq f(1002)\leq 1002^2=1004004,\] lo que deja relativamente pocos valores posibles para $f(1002)$. Ahora bien, la segunda condición nos da también $f(2)\leq 4$, $f(3)\leq 9$ y $f(167)\leq 27889$. Vamos a ver que estas tres desigualdades tienen que ser igualdades, pues en caso contrario, se tendría que $f(2)\leq 3$, $f(3)\leq 8$ o $f(167)\leq 27888$, luego se cumpliría alguna de las siguientes desigualdades \begin{align*} f(1002)&=f(2)f(3)f(167)\leq 4\cdot 9\cdot 27888=1003968,\\ f(1002)&=f(2)f(3)f(167)\leq 4\cdot 8\cdot 27889=892448,\\ f(1002)&=f(2)f(3)f(167)\leq 3\cdot 9\cdot 27889=753003, \end{align*} pero ninguna de ellas es posible ya que $f(1002)\geq 1003969$ (en este punto nos damos cuenta de por qué se ha puesto ese número raro en el enunciado). Deducimos, pues, que \[f(2004)=f(2)^2f(3)f(167)=4^2\cdot 9\cdot 27889=2004^2=4016016\] es el único posible valor.
Si crees que el enunciado contiene un error o imprecisión o bien crees que la información sobre la procedencia del problema es incorrecta, puedes notificarlo usando los siguientes botones:
Informar de error en enunciado Informar de procedencia del problema
Problema 702
Consideramos los polinomios $P(x)=x^3+Ax^2+Bx+C$ y $Q(x)=3x^2+2Ax+B$. Supongamos que las dos raíces de $Q(x)$ son $\frac{a+b}{2}$ y $\frac{b+c}{2}$, siendo $a,b,c$ las tres raíces de $P(x)$. Determinar todos los posibles polinomios $P(x)$ y $Q(x)$ que cumplen esta condición.
pistasolución 1info
Pista. Utiliza las relaciones de Cardano-Vieta y las relaciones entre los coeficientes de $P(x)$ y $Q(x)$.
Solución. Como sabemos las raíces de ambos polinomios, podemos escribir \begin{align*} P(x)&=x^3+Ax^2+Bx+C=(x-a)(x-b)(x-c),\\ Q(x)&=3x^2+2Ax+B=3(x-\tfrac{a+b}{2})(x-\tfrac{b+c}{2}). \end{align*} Desarrollando los paréntesis e igualando coeficientes, obtenemos las llamadas relaciones de Cardano-Vieta para ambos polinomios: \[P(x):\left\{\begin{array}{l} a+b+c=-A\\ ab+bc+ac=B\\ abc=-C \end{array}\right.\qquad\qquad Q(x):\left\{\begin{array}{l}a+2b+c=\frac{-4A}{3}\\ ab+bc+ca+b^2=\frac{4B}{3}\end{array}\right.\] Usando las relaciones para $P(x)$, podemos transformar las de $Q(x)$ en \[\tfrac{-4A}{3}=a+2b+c=-A+b,\qquad \tfrac{4B}{3}=ab+bc+ca+b^2=B+b^2,\] de donde deducimos que $b=\frac{-A}{3}$ y $b^2=\frac{B}{3}$. De aquí podemos decir que $\frac{A^2}{9}=b^2=\frac{B}{3}$, luego $A^2=3B$, lo que a su vez nos permite reescribir \[Q(x)=3x^2+2Ax+B=3x^2+2Ax+\tfrac{1}{3}A^2=\frac{1}{3}(3x+A)^2.\] Por tanto, las dos raíces de $Q(x)$ son iguales a $\frac{-A}{3}$, es decir, \[\frac{a+b}{2}=\frac{b+c}{2}=\frac{-A}{3}\ \Longleftrightarrow\ a=c=\frac{-2A}{3}-b=\frac{-A}{3}.\] Como ya sabíamos que $b=\frac{-A}{3}$, llegamos a que las tres raíces de $P(x)$ tienen que ser iguales, es decir, la solución al problema son los polinomios $P(x)=(x-a)^3$ y $Q(x)=3(x-a)^2$ para cualquier $a\in\mathbb{R}$, que claramente cumplen las condiciones propuestas.

Nota. Este resultado es curioso porque si pensamos en un polinomio $P(x)$ con tres raíces reales $a\lt b\lt c$, el polinomio $Q(x)$ es la derivada de $P(x)$ y tiene una raíz estrictamente entre $a$ y $b$ y la otra estrictamente entre $b$ y $c$ (por el teorema de Rolle). El problema nos dice que, salvo que todas las raíces coincidan, el máximo y el mínimo locales de $P(x)$ no pueden ser los puntos medios de los intervalos $[a,b]$ y $[b,c]$. En realidad, se puede demostrar que siempre estas raíces están más cerca de $a$ o $c$ que de $b$. ¿Sabrías probarlo?

Si crees que el enunciado contiene un error o imprecisión o bien crees que la información sobre la procedencia del problema es incorrecta, puedes notificarlo usando los siguientes botones:
Informar de error en enunciado Informar de procedencia del problema
Problema 701
Un triángulo tiene un vértice en cada uno de los ejes de un sistema de coordenadas cartesianas en el espacio y ningún vértice está en el origen. Demostrar que el triángulo es acutángulo.
pistasolución 1info
Pista. Los vértices tienen coordenadas $A=(a,0,0)$, $B=(0,b,0)$ y $C=(0,0,c)$. Los ángulos se pueden ahora calcular mediante la fórmula para el ángulo de dos vectores en el espacio.
Solución. Pongamos que los vértices están dados en coordenadas por $A=(a,0,0)$, $B=(0,b,0)$ y $C=(0,0,c)$, siendo $a,b,c\neq 0$. Vamos a calcular el ángulo en el vértice $A$ usando geometría analítica (los otros dos ángulos se razonan de forma similar). Considerando los vectores directores $\vec{u}=(-a,b,0)$ del lado $AB$ y $\vec{v}=(-a,0,c)$ del lado $BC$, tenemos que \[\cos(A)=\frac{\vec{u}\cdot \vec{v}}{\|\vec{u}\|\cdot\|\vec{v}\|}=\frac{a^2}{\sqrt{a^2+b^2}\sqrt{a^2+c^2}}\gt 0.\] Por tanto, deducimos que $A\lt 90^\circ$.
Si crees que el enunciado contiene un error o imprecisión o bien crees que la información sobre la procedencia del problema es incorrecta, puedes notificarlo usando los siguientes botones:
Informar de error en enunciado Informar de procedencia del problema
Problema 700
Hallar todas las funciones $f:\mathbb{N}\to\mathbb{N}$ tales que $f(f(n))=n+2$ para todo número natural $n\in\mathbb{N}$.
pistasolución 1info
Pista. Calcula $f(f(f(n)))$ de dos maneras distintas.
Solución. Si en la ecuación sustituimos $n$ por $f(n)$, tenemos que $f(f(f(n)))=f(n)+2$. Por otro lado, si aplicamos $f$ a la ecuación también tenemos que $f(f(f(n)))=f(n+2)$, por lo que la función $f$ tiene que verificar \[f(n+2)=f(n)+2,\quad\text{para todo }n\in\mathbb{N}.\] Esto nos dice que $f(n)$ avanza dos unidades cuando $n$ aumenta dos unidades, luego crece linealmente en los pares y en los impares por separado. Si llamamos $a=f(1)$ y $b=f(2)$, se tiene que \[f(n)=\begin{cases}a+n-1&\text{si }n\text{ es impar},\\ b+n-2&\text{si }n\text{ es par}.\end{cases}\] Queda por ver qué valores de $a$ y $b$ hacen que se cumpla la ecuación inicial, para lo que distinguiremos casos según la paridad de $a$ y $b$:
  • Si $a$ es impar, entonces $n+2=f(f(n))=f(a+n-1)=2a+n-2$ para todo $n$ impar, luego tiene que ser $2a-2=2$, que no es impar. De la misma forma, si $b$ es par, entonces $n+2=f(f(n))=f(b+n-2)=2b+n-4$ para todo $n$ par, luego tiene que ser $2b-4=2$, es decir, $b=3$, que no es par.
  • Tenemos entonces que $a$ es par y $b$ impar, luego $n+2=f(f(n))=f(a+n-1)=a+b+n-3$ para todo $n$ impar, luego tiene que ser $a+b=5$, cuya única posible solución es $a=2$ y $b=3$.
Deducimos así que la única solución a la ecuación funcional es $f(n)=n+1$ para todo $n\in\mathbb{N}$.

Nota. Se ha considerado que los naturales no contienen al cero. En caso de que el cero sea admitido como natural, la ecuación $a+b=5$ del último punto también admite la solución $a=0$ y $b=5$ (recordemos que $a$ y $b$ son valores de $f$). Esto nos daría lugar a la solución \[f(n)=\begin{cases}n-1&\text{si }n\text{ es impar},\\ n+3&\text{si }n\text{ es par}.\end{cases}\]

Si crees que el enunciado contiene un error o imprecisión o bien crees que la información sobre la procedencia del problema es incorrecta, puedes notificarlo usando los siguientes botones:
Informar de error en enunciado Informar de procedencia del problema
José Miguel Manzano © 2010-2024. Esta página ha sido creada mediante software libre